LSAT and Law School Admissions Forum

Get expert LSAT preparation and law school admissions advice from PowerScore Test Preparation.

User avatar
 Dave Killoran
PowerScore Staff
  • PowerScore Staff
  • Posts: 5972
  • Joined: Mar 25, 2011
|
#85537
Complete Question Explanation
(The complete setup for this game can be found here: lsat/viewtopic.php?f=307&t=9348)

The correct answer choice is (C)

This is a very difficult List question.

Answer choice (A) is incorrect because it violates the fourth rule.

Answer choice (B) is incorrect because it would violate the second rule: as discussed during the setup, because of that rule, J can never be inspected on Monday morning.

Answer choice (C) is the correct answer. This is not a violation of the fourth rule: as discussed during the setup, L can be inspected in the morning and Z can be inspected in the afternoon.

Answer choice (D) is incorrect because it cause a violation of the third rule: when J is inspected on Tuesday morning, then from the second rule G must be inspected on Monday, but with S already inspected on Monday, a violation ensues.

Answer choice (E) is incorrect because it ultimately causes a violation of the third rule. With S, L, and Z as the morning inspections, only V remains to be inspected on Wednesday afternoon. At that point, 4 of the 6 buildings have been inspected, leaving only G and J. Due to the second rule, G must then be inspected on Monday afternoon, and J on Tuesday afternoon. But, that results in G and S both being inspected on Monday, a violation of the third rule.
 NR 2020
  • Posts: 5
  • Joined: Oct 04, 2020
|
#80731
Can you please post explanation for question 14?
 Jeremy Press
PowerScore Staff
  • PowerScore Staff
  • Posts: 1000
  • Joined: Jun 12, 2017
|
#80751
Hi NR2020,

In this Global, Partial List question, you should begin by looking through the answer choices for obvious rule violations. Take each rule in order, and look to see if it is directly, or indirectly, violated in the answer choices.

The first rule is indirectly violated in answer choice E. Because G and S cannot be inspected on the same day, and because hotels cannot be inspected on Wednesday, G would have to be inspected on Tuesday afternoon. This would force J to be inspected on Wednesday, which violates the first rule that hotels cannot be inspected on Wednesday. Thus, you can eliminate answer choice E.

The second rule is indirectly violated in answer choice B. Because J has to be inspected at some time after G, J cannot be inspected in the first available inspection time (Monday morning). So, eliminate answer choice B.

The third rule is indirectly violated in answer choice D. Because J is on Tuesday morning, that would force G onto Monday. With S already on Monday morning in that answer choice, that would force G into Monday afternoon. Since G and S cannot be inspected on the same day, you can eliminate answer choice D.

The fourth rule is directly violated in answer choice A. Because Z is inspected in the morning, L should also be inspected in the morning. Since L is not inspected in the morning, you can eliminate answer choice A.

This process of elimination leaves answer choice C as the correct answer.

I hope this helps!

Get the most out of your LSAT Prep Plus subscription.

Analyze and track your performance with our Testing and Analytics Package.